Calculus of variations confusion











up vote
2
down vote

favorite
1












Let's say I have the lagrangian
$$L(x,u,u')=u'$$
If apply the Euler-Lagrange equation for $L$, I have:
$$frac{partial L}{partial u}-frac{d}{dx}frac{partial L}{partial u'}=0-frac{d}{dx}(1)=0$$
If I choose to apply to $L^2$, I have:



begin{equation}
frac{partial L^2}{partial u}-frac{d}{dx}frac{partial L^2}{partial u'}=0-frac{d}{dx}(2u')=-2u'' tag{1}
end{equation}



If I expand the Euler-Langrange equation for $L^2$, I get:



begin{equation}
frac{partial L^2}{partial u}-frac{d}{dx}frac{partial L^2}{partial u'}=2Lfrac{partial L}{partial u}-frac{d}{dx}(2Lfrac{partial L}{partial u'})=2L(frac{partial L}{partial u}-frac{d}{dx}frac{partial L}{partial u'})-2frac{dL}{dx}frac{partial L}{partial u'}
end{equation}



$L$ does not explicitly depend on x, so:



begin{equation}
frac{partial L^2}{partial u}-frac{d}{dx}frac{partial L^2}{partial u'}=2L(frac{partial L}{partial u}-frac{d}{dx}frac{partial L}{partial u'})=2L(0)=0?? tag{2}
end{equation}



Why does $(1)$ differ from $(2)$? What am I doing wrong?










share|cite|improve this question




























    up vote
    2
    down vote

    favorite
    1












    Let's say I have the lagrangian
    $$L(x,u,u')=u'$$
    If apply the Euler-Lagrange equation for $L$, I have:
    $$frac{partial L}{partial u}-frac{d}{dx}frac{partial L}{partial u'}=0-frac{d}{dx}(1)=0$$
    If I choose to apply to $L^2$, I have:



    begin{equation}
    frac{partial L^2}{partial u}-frac{d}{dx}frac{partial L^2}{partial u'}=0-frac{d}{dx}(2u')=-2u'' tag{1}
    end{equation}



    If I expand the Euler-Langrange equation for $L^2$, I get:



    begin{equation}
    frac{partial L^2}{partial u}-frac{d}{dx}frac{partial L^2}{partial u'}=2Lfrac{partial L}{partial u}-frac{d}{dx}(2Lfrac{partial L}{partial u'})=2L(frac{partial L}{partial u}-frac{d}{dx}frac{partial L}{partial u'})-2frac{dL}{dx}frac{partial L}{partial u'}
    end{equation}



    $L$ does not explicitly depend on x, so:



    begin{equation}
    frac{partial L^2}{partial u}-frac{d}{dx}frac{partial L^2}{partial u'}=2L(frac{partial L}{partial u}-frac{d}{dx}frac{partial L}{partial u'})=2L(0)=0?? tag{2}
    end{equation}



    Why does $(1)$ differ from $(2)$? What am I doing wrong?










    share|cite|improve this question


























      up vote
      2
      down vote

      favorite
      1









      up vote
      2
      down vote

      favorite
      1






      1





      Let's say I have the lagrangian
      $$L(x,u,u')=u'$$
      If apply the Euler-Lagrange equation for $L$, I have:
      $$frac{partial L}{partial u}-frac{d}{dx}frac{partial L}{partial u'}=0-frac{d}{dx}(1)=0$$
      If I choose to apply to $L^2$, I have:



      begin{equation}
      frac{partial L^2}{partial u}-frac{d}{dx}frac{partial L^2}{partial u'}=0-frac{d}{dx}(2u')=-2u'' tag{1}
      end{equation}



      If I expand the Euler-Langrange equation for $L^2$, I get:



      begin{equation}
      frac{partial L^2}{partial u}-frac{d}{dx}frac{partial L^2}{partial u'}=2Lfrac{partial L}{partial u}-frac{d}{dx}(2Lfrac{partial L}{partial u'})=2L(frac{partial L}{partial u}-frac{d}{dx}frac{partial L}{partial u'})-2frac{dL}{dx}frac{partial L}{partial u'}
      end{equation}



      $L$ does not explicitly depend on x, so:



      begin{equation}
      frac{partial L^2}{partial u}-frac{d}{dx}frac{partial L^2}{partial u'}=2L(frac{partial L}{partial u}-frac{d}{dx}frac{partial L}{partial u'})=2L(0)=0?? tag{2}
      end{equation}



      Why does $(1)$ differ from $(2)$? What am I doing wrong?










      share|cite|improve this question















      Let's say I have the lagrangian
      $$L(x,u,u')=u'$$
      If apply the Euler-Lagrange equation for $L$, I have:
      $$frac{partial L}{partial u}-frac{d}{dx}frac{partial L}{partial u'}=0-frac{d}{dx}(1)=0$$
      If I choose to apply to $L^2$, I have:



      begin{equation}
      frac{partial L^2}{partial u}-frac{d}{dx}frac{partial L^2}{partial u'}=0-frac{d}{dx}(2u')=-2u'' tag{1}
      end{equation}



      If I expand the Euler-Langrange equation for $L^2$, I get:



      begin{equation}
      frac{partial L^2}{partial u}-frac{d}{dx}frac{partial L^2}{partial u'}=2Lfrac{partial L}{partial u}-frac{d}{dx}(2Lfrac{partial L}{partial u'})=2L(frac{partial L}{partial u}-frac{d}{dx}frac{partial L}{partial u'})-2frac{dL}{dx}frac{partial L}{partial u'}
      end{equation}



      $L$ does not explicitly depend on x, so:



      begin{equation}
      frac{partial L^2}{partial u}-frac{d}{dx}frac{partial L^2}{partial u'}=2L(frac{partial L}{partial u}-frac{d}{dx}frac{partial L}{partial u'})=2L(0)=0?? tag{2}
      end{equation}



      Why does $(1)$ differ from $(2)$? What am I doing wrong?







      calculus multivariable-calculus calculus-of-variations






      share|cite|improve this question















      share|cite|improve this question













      share|cite|improve this question




      share|cite|improve this question








      edited Nov 24 at 10:55

























      asked Nov 24 at 9:15









      BinaryBurst

      359110




      359110






















          2 Answers
          2






          active

          oldest

          votes

















          up vote
          1
          down vote



          accepted










          There are several issues to consider here:




          • In general, changing a choice of Lagrangian from $L$ to $L^2$ doesn't preserve the equations of motion. For example, $L=frac{1}{2}u'^2-V(u)implies u''=-partial_u V$, but $$L=frac{1}{4}u'^4-u'^2 V(u) + V^2(u)implies u''=-frac{2V+u'^2}{2V-3u'^2}partial_uV.$$These are only equivalent if $u'=0$.

          • Probably the most famous example where squaring a Lagrangian does preserve the EOMs is $L=sqrt{g_{munu}partial_sx^mupartial_sx^nu}$ when finding the geodesic deviation equation. Your comment on an existing answer suggests you thought this generalises. It does not. Even in the geodesic example, the only sense in which the two approaches are equivalent is that you can choose an affine parameter so as to equate the results. See the accepted answer to this existing question.

          • Your original Lagrangian $u'$ is a total derivative, with the same vacuously true Euler-Lagrange equation as the choice $L=0$.

          • In $L=u'$, $u$ has momentum $p=1$. This is a phase space constraint, which opens a can of worms. For example, if we tried to obtain the Hamiltonian for this Lagrangian, you'd just get $0$ until you include Dirac brackets viz. $H=f(u,,p)(p-1)$ for some function $f$. This gives the Hamilton's equations $u'=f+(p-1)partial_p f,,0=p'=(1-p)partial_u f$. The problem is we can't solve this for $f$.

          • The Lagrangian $u'^2$ has equation of motion $u''=0$ anyway.






          share|cite|improve this answer





















          • I was wondering why I get different answers for the $L^2$ lagrangian by plugging it in directly into the Euler-Lagrange equation and computing versus by expanding the Euler-Lagrange equations and then plugging in for $L$. Shouldn't the answers be the same, because $L$ doesn't explicitly depend on x? After all, it's the same equation in two forms. I apologise for not stating the question clearly enough.
            – BinaryBurst
            Nov 24 at 10:26












          • I'm guessing that I do have to keep the $frac{dL}{dx}$ and carry it out even though $L$ doesn't explicitly depend on x.
            – BinaryBurst
            Nov 24 at 10:34








          • 1




            @BinaryBurst Since total derivatives aren't the same as partial ones, $frac{dL}{dx}=frac{partial L}{partial u'}u''=u''$.
            – J.G.
            Nov 24 at 10:41










          • So are you saying $L_1 land L_2$ are the same insofar as that the value is concerned, but not the fuctions? $L_1=L_2^2=l(x,u,u')=alpha$ For instance, in this case, we can equate $L_1$ and $L_2$ by the value $alpha$, but not $l(x,u,u')$, Right? Or am I missing something?
            – Bertrand Wittgenstein's Ghost
            Nov 24 at 12:10


















          up vote
          0
          down vote













          Let me define these two Lagrangians



          $$
          L_1 = u' tag{a}
          $$



          and



          $$
          L_2 = (u')^2 tag{b}
          $$



          As it turns out $L_1$ has its own equations of motion and they do not need to satisfy equation (b). In other words, a solution of Eq. (a) it is not the same as a solution of Eq. (b). With this in mind, the problem with your argument is that in your equation (1) you assume that the Lagrangian is $L_2 = L^2$, and in your equation (2) you then assume that the Lagrangian is $L_1 = L$, which clearly will lead to a problem!






          share|cite|improve this answer





















          • The lagrangian for a geodesic is $sqrt{g_{ij}frac{dx^i}{ds}frac{dx^j}{ds}}$ but they use the square of this quantity as the lagrangian to derive the equations of motion. Shouldn't that be a problem?
            – BinaryBurst
            Nov 24 at 10:09












          • @BinaryBurst Was editing my answer when I saw J.G's
            – caverac
            Nov 24 at 10:20











          Your Answer





          StackExchange.ifUsing("editor", function () {
          return StackExchange.using("mathjaxEditing", function () {
          StackExchange.MarkdownEditor.creationCallbacks.add(function (editor, postfix) {
          StackExchange.mathjaxEditing.prepareWmdForMathJax(editor, postfix, [["$", "$"], ["\\(","\\)"]]);
          });
          });
          }, "mathjax-editing");

          StackExchange.ready(function() {
          var channelOptions = {
          tags: "".split(" "),
          id: "69"
          };
          initTagRenderer("".split(" "), "".split(" "), channelOptions);

          StackExchange.using("externalEditor", function() {
          // Have to fire editor after snippets, if snippets enabled
          if (StackExchange.settings.snippets.snippetsEnabled) {
          StackExchange.using("snippets", function() {
          createEditor();
          });
          }
          else {
          createEditor();
          }
          });

          function createEditor() {
          StackExchange.prepareEditor({
          heartbeatType: 'answer',
          autoActivateHeartbeat: false,
          convertImagesToLinks: true,
          noModals: true,
          showLowRepImageUploadWarning: true,
          reputationToPostImages: 10,
          bindNavPrevention: true,
          postfix: "",
          imageUploader: {
          brandingHtml: "Powered by u003ca class="icon-imgur-white" href="https://imgur.com/"u003eu003c/au003e",
          contentPolicyHtml: "User contributions licensed under u003ca href="https://creativecommons.org/licenses/by-sa/3.0/"u003ecc by-sa 3.0 with attribution requiredu003c/au003e u003ca href="https://stackoverflow.com/legal/content-policy"u003e(content policy)u003c/au003e",
          allowUrls: true
          },
          noCode: true, onDemand: true,
          discardSelector: ".discard-answer"
          ,immediatelyShowMarkdownHelp:true
          });


          }
          });














          draft saved

          draft discarded


















          StackExchange.ready(
          function () {
          StackExchange.openid.initPostLogin('.new-post-login', 'https%3a%2f%2fmath.stackexchange.com%2fquestions%2f3011349%2fcalculus-of-variations-confusion%23new-answer', 'question_page');
          }
          );

          Post as a guest















          Required, but never shown

























          2 Answers
          2






          active

          oldest

          votes








          2 Answers
          2






          active

          oldest

          votes









          active

          oldest

          votes






          active

          oldest

          votes








          up vote
          1
          down vote



          accepted










          There are several issues to consider here:




          • In general, changing a choice of Lagrangian from $L$ to $L^2$ doesn't preserve the equations of motion. For example, $L=frac{1}{2}u'^2-V(u)implies u''=-partial_u V$, but $$L=frac{1}{4}u'^4-u'^2 V(u) + V^2(u)implies u''=-frac{2V+u'^2}{2V-3u'^2}partial_uV.$$These are only equivalent if $u'=0$.

          • Probably the most famous example where squaring a Lagrangian does preserve the EOMs is $L=sqrt{g_{munu}partial_sx^mupartial_sx^nu}$ when finding the geodesic deviation equation. Your comment on an existing answer suggests you thought this generalises. It does not. Even in the geodesic example, the only sense in which the two approaches are equivalent is that you can choose an affine parameter so as to equate the results. See the accepted answer to this existing question.

          • Your original Lagrangian $u'$ is a total derivative, with the same vacuously true Euler-Lagrange equation as the choice $L=0$.

          • In $L=u'$, $u$ has momentum $p=1$. This is a phase space constraint, which opens a can of worms. For example, if we tried to obtain the Hamiltonian for this Lagrangian, you'd just get $0$ until you include Dirac brackets viz. $H=f(u,,p)(p-1)$ for some function $f$. This gives the Hamilton's equations $u'=f+(p-1)partial_p f,,0=p'=(1-p)partial_u f$. The problem is we can't solve this for $f$.

          • The Lagrangian $u'^2$ has equation of motion $u''=0$ anyway.






          share|cite|improve this answer





















          • I was wondering why I get different answers for the $L^2$ lagrangian by plugging it in directly into the Euler-Lagrange equation and computing versus by expanding the Euler-Lagrange equations and then plugging in for $L$. Shouldn't the answers be the same, because $L$ doesn't explicitly depend on x? After all, it's the same equation in two forms. I apologise for not stating the question clearly enough.
            – BinaryBurst
            Nov 24 at 10:26












          • I'm guessing that I do have to keep the $frac{dL}{dx}$ and carry it out even though $L$ doesn't explicitly depend on x.
            – BinaryBurst
            Nov 24 at 10:34








          • 1




            @BinaryBurst Since total derivatives aren't the same as partial ones, $frac{dL}{dx}=frac{partial L}{partial u'}u''=u''$.
            – J.G.
            Nov 24 at 10:41










          • So are you saying $L_1 land L_2$ are the same insofar as that the value is concerned, but not the fuctions? $L_1=L_2^2=l(x,u,u')=alpha$ For instance, in this case, we can equate $L_1$ and $L_2$ by the value $alpha$, but not $l(x,u,u')$, Right? Or am I missing something?
            – Bertrand Wittgenstein's Ghost
            Nov 24 at 12:10















          up vote
          1
          down vote



          accepted










          There are several issues to consider here:




          • In general, changing a choice of Lagrangian from $L$ to $L^2$ doesn't preserve the equations of motion. For example, $L=frac{1}{2}u'^2-V(u)implies u''=-partial_u V$, but $$L=frac{1}{4}u'^4-u'^2 V(u) + V^2(u)implies u''=-frac{2V+u'^2}{2V-3u'^2}partial_uV.$$These are only equivalent if $u'=0$.

          • Probably the most famous example where squaring a Lagrangian does preserve the EOMs is $L=sqrt{g_{munu}partial_sx^mupartial_sx^nu}$ when finding the geodesic deviation equation. Your comment on an existing answer suggests you thought this generalises. It does not. Even in the geodesic example, the only sense in which the two approaches are equivalent is that you can choose an affine parameter so as to equate the results. See the accepted answer to this existing question.

          • Your original Lagrangian $u'$ is a total derivative, with the same vacuously true Euler-Lagrange equation as the choice $L=0$.

          • In $L=u'$, $u$ has momentum $p=1$. This is a phase space constraint, which opens a can of worms. For example, if we tried to obtain the Hamiltonian for this Lagrangian, you'd just get $0$ until you include Dirac brackets viz. $H=f(u,,p)(p-1)$ for some function $f$. This gives the Hamilton's equations $u'=f+(p-1)partial_p f,,0=p'=(1-p)partial_u f$. The problem is we can't solve this for $f$.

          • The Lagrangian $u'^2$ has equation of motion $u''=0$ anyway.






          share|cite|improve this answer





















          • I was wondering why I get different answers for the $L^2$ lagrangian by plugging it in directly into the Euler-Lagrange equation and computing versus by expanding the Euler-Lagrange equations and then plugging in for $L$. Shouldn't the answers be the same, because $L$ doesn't explicitly depend on x? After all, it's the same equation in two forms. I apologise for not stating the question clearly enough.
            – BinaryBurst
            Nov 24 at 10:26












          • I'm guessing that I do have to keep the $frac{dL}{dx}$ and carry it out even though $L$ doesn't explicitly depend on x.
            – BinaryBurst
            Nov 24 at 10:34








          • 1




            @BinaryBurst Since total derivatives aren't the same as partial ones, $frac{dL}{dx}=frac{partial L}{partial u'}u''=u''$.
            – J.G.
            Nov 24 at 10:41










          • So are you saying $L_1 land L_2$ are the same insofar as that the value is concerned, but not the fuctions? $L_1=L_2^2=l(x,u,u')=alpha$ For instance, in this case, we can equate $L_1$ and $L_2$ by the value $alpha$, but not $l(x,u,u')$, Right? Or am I missing something?
            – Bertrand Wittgenstein's Ghost
            Nov 24 at 12:10













          up vote
          1
          down vote



          accepted







          up vote
          1
          down vote



          accepted






          There are several issues to consider here:




          • In general, changing a choice of Lagrangian from $L$ to $L^2$ doesn't preserve the equations of motion. For example, $L=frac{1}{2}u'^2-V(u)implies u''=-partial_u V$, but $$L=frac{1}{4}u'^4-u'^2 V(u) + V^2(u)implies u''=-frac{2V+u'^2}{2V-3u'^2}partial_uV.$$These are only equivalent if $u'=0$.

          • Probably the most famous example where squaring a Lagrangian does preserve the EOMs is $L=sqrt{g_{munu}partial_sx^mupartial_sx^nu}$ when finding the geodesic deviation equation. Your comment on an existing answer suggests you thought this generalises. It does not. Even in the geodesic example, the only sense in which the two approaches are equivalent is that you can choose an affine parameter so as to equate the results. See the accepted answer to this existing question.

          • Your original Lagrangian $u'$ is a total derivative, with the same vacuously true Euler-Lagrange equation as the choice $L=0$.

          • In $L=u'$, $u$ has momentum $p=1$. This is a phase space constraint, which opens a can of worms. For example, if we tried to obtain the Hamiltonian for this Lagrangian, you'd just get $0$ until you include Dirac brackets viz. $H=f(u,,p)(p-1)$ for some function $f$. This gives the Hamilton's equations $u'=f+(p-1)partial_p f,,0=p'=(1-p)partial_u f$. The problem is we can't solve this for $f$.

          • The Lagrangian $u'^2$ has equation of motion $u''=0$ anyway.






          share|cite|improve this answer












          There are several issues to consider here:




          • In general, changing a choice of Lagrangian from $L$ to $L^2$ doesn't preserve the equations of motion. For example, $L=frac{1}{2}u'^2-V(u)implies u''=-partial_u V$, but $$L=frac{1}{4}u'^4-u'^2 V(u) + V^2(u)implies u''=-frac{2V+u'^2}{2V-3u'^2}partial_uV.$$These are only equivalent if $u'=0$.

          • Probably the most famous example where squaring a Lagrangian does preserve the EOMs is $L=sqrt{g_{munu}partial_sx^mupartial_sx^nu}$ when finding the geodesic deviation equation. Your comment on an existing answer suggests you thought this generalises. It does not. Even in the geodesic example, the only sense in which the two approaches are equivalent is that you can choose an affine parameter so as to equate the results. See the accepted answer to this existing question.

          • Your original Lagrangian $u'$ is a total derivative, with the same vacuously true Euler-Lagrange equation as the choice $L=0$.

          • In $L=u'$, $u$ has momentum $p=1$. This is a phase space constraint, which opens a can of worms. For example, if we tried to obtain the Hamiltonian for this Lagrangian, you'd just get $0$ until you include Dirac brackets viz. $H=f(u,,p)(p-1)$ for some function $f$. This gives the Hamilton's equations $u'=f+(p-1)partial_p f,,0=p'=(1-p)partial_u f$. The problem is we can't solve this for $f$.

          • The Lagrangian $u'^2$ has equation of motion $u''=0$ anyway.







          share|cite|improve this answer












          share|cite|improve this answer



          share|cite|improve this answer










          answered Nov 24 at 10:15









          J.G.

          21.3k21933




          21.3k21933












          • I was wondering why I get different answers for the $L^2$ lagrangian by plugging it in directly into the Euler-Lagrange equation and computing versus by expanding the Euler-Lagrange equations and then plugging in for $L$. Shouldn't the answers be the same, because $L$ doesn't explicitly depend on x? After all, it's the same equation in two forms. I apologise for not stating the question clearly enough.
            – BinaryBurst
            Nov 24 at 10:26












          • I'm guessing that I do have to keep the $frac{dL}{dx}$ and carry it out even though $L$ doesn't explicitly depend on x.
            – BinaryBurst
            Nov 24 at 10:34








          • 1




            @BinaryBurst Since total derivatives aren't the same as partial ones, $frac{dL}{dx}=frac{partial L}{partial u'}u''=u''$.
            – J.G.
            Nov 24 at 10:41










          • So are you saying $L_1 land L_2$ are the same insofar as that the value is concerned, but not the fuctions? $L_1=L_2^2=l(x,u,u')=alpha$ For instance, in this case, we can equate $L_1$ and $L_2$ by the value $alpha$, but not $l(x,u,u')$, Right? Or am I missing something?
            – Bertrand Wittgenstein's Ghost
            Nov 24 at 12:10


















          • I was wondering why I get different answers for the $L^2$ lagrangian by plugging it in directly into the Euler-Lagrange equation and computing versus by expanding the Euler-Lagrange equations and then plugging in for $L$. Shouldn't the answers be the same, because $L$ doesn't explicitly depend on x? After all, it's the same equation in two forms. I apologise for not stating the question clearly enough.
            – BinaryBurst
            Nov 24 at 10:26












          • I'm guessing that I do have to keep the $frac{dL}{dx}$ and carry it out even though $L$ doesn't explicitly depend on x.
            – BinaryBurst
            Nov 24 at 10:34








          • 1




            @BinaryBurst Since total derivatives aren't the same as partial ones, $frac{dL}{dx}=frac{partial L}{partial u'}u''=u''$.
            – J.G.
            Nov 24 at 10:41










          • So are you saying $L_1 land L_2$ are the same insofar as that the value is concerned, but not the fuctions? $L_1=L_2^2=l(x,u,u')=alpha$ For instance, in this case, we can equate $L_1$ and $L_2$ by the value $alpha$, but not $l(x,u,u')$, Right? Or am I missing something?
            – Bertrand Wittgenstein's Ghost
            Nov 24 at 12:10
















          I was wondering why I get different answers for the $L^2$ lagrangian by plugging it in directly into the Euler-Lagrange equation and computing versus by expanding the Euler-Lagrange equations and then plugging in for $L$. Shouldn't the answers be the same, because $L$ doesn't explicitly depend on x? After all, it's the same equation in two forms. I apologise for not stating the question clearly enough.
          – BinaryBurst
          Nov 24 at 10:26






          I was wondering why I get different answers for the $L^2$ lagrangian by plugging it in directly into the Euler-Lagrange equation and computing versus by expanding the Euler-Lagrange equations and then plugging in for $L$. Shouldn't the answers be the same, because $L$ doesn't explicitly depend on x? After all, it's the same equation in two forms. I apologise for not stating the question clearly enough.
          – BinaryBurst
          Nov 24 at 10:26














          I'm guessing that I do have to keep the $frac{dL}{dx}$ and carry it out even though $L$ doesn't explicitly depend on x.
          – BinaryBurst
          Nov 24 at 10:34






          I'm guessing that I do have to keep the $frac{dL}{dx}$ and carry it out even though $L$ doesn't explicitly depend on x.
          – BinaryBurst
          Nov 24 at 10:34






          1




          1




          @BinaryBurst Since total derivatives aren't the same as partial ones, $frac{dL}{dx}=frac{partial L}{partial u'}u''=u''$.
          – J.G.
          Nov 24 at 10:41




          @BinaryBurst Since total derivatives aren't the same as partial ones, $frac{dL}{dx}=frac{partial L}{partial u'}u''=u''$.
          – J.G.
          Nov 24 at 10:41












          So are you saying $L_1 land L_2$ are the same insofar as that the value is concerned, but not the fuctions? $L_1=L_2^2=l(x,u,u')=alpha$ For instance, in this case, we can equate $L_1$ and $L_2$ by the value $alpha$, but not $l(x,u,u')$, Right? Or am I missing something?
          – Bertrand Wittgenstein's Ghost
          Nov 24 at 12:10




          So are you saying $L_1 land L_2$ are the same insofar as that the value is concerned, but not the fuctions? $L_1=L_2^2=l(x,u,u')=alpha$ For instance, in this case, we can equate $L_1$ and $L_2$ by the value $alpha$, but not $l(x,u,u')$, Right? Or am I missing something?
          – Bertrand Wittgenstein's Ghost
          Nov 24 at 12:10










          up vote
          0
          down vote













          Let me define these two Lagrangians



          $$
          L_1 = u' tag{a}
          $$



          and



          $$
          L_2 = (u')^2 tag{b}
          $$



          As it turns out $L_1$ has its own equations of motion and they do not need to satisfy equation (b). In other words, a solution of Eq. (a) it is not the same as a solution of Eq. (b). With this in mind, the problem with your argument is that in your equation (1) you assume that the Lagrangian is $L_2 = L^2$, and in your equation (2) you then assume that the Lagrangian is $L_1 = L$, which clearly will lead to a problem!






          share|cite|improve this answer





















          • The lagrangian for a geodesic is $sqrt{g_{ij}frac{dx^i}{ds}frac{dx^j}{ds}}$ but they use the square of this quantity as the lagrangian to derive the equations of motion. Shouldn't that be a problem?
            – BinaryBurst
            Nov 24 at 10:09












          • @BinaryBurst Was editing my answer when I saw J.G's
            – caverac
            Nov 24 at 10:20















          up vote
          0
          down vote













          Let me define these two Lagrangians



          $$
          L_1 = u' tag{a}
          $$



          and



          $$
          L_2 = (u')^2 tag{b}
          $$



          As it turns out $L_1$ has its own equations of motion and they do not need to satisfy equation (b). In other words, a solution of Eq. (a) it is not the same as a solution of Eq. (b). With this in mind, the problem with your argument is that in your equation (1) you assume that the Lagrangian is $L_2 = L^2$, and in your equation (2) you then assume that the Lagrangian is $L_1 = L$, which clearly will lead to a problem!






          share|cite|improve this answer





















          • The lagrangian for a geodesic is $sqrt{g_{ij}frac{dx^i}{ds}frac{dx^j}{ds}}$ but they use the square of this quantity as the lagrangian to derive the equations of motion. Shouldn't that be a problem?
            – BinaryBurst
            Nov 24 at 10:09












          • @BinaryBurst Was editing my answer when I saw J.G's
            – caverac
            Nov 24 at 10:20













          up vote
          0
          down vote










          up vote
          0
          down vote









          Let me define these two Lagrangians



          $$
          L_1 = u' tag{a}
          $$



          and



          $$
          L_2 = (u')^2 tag{b}
          $$



          As it turns out $L_1$ has its own equations of motion and they do not need to satisfy equation (b). In other words, a solution of Eq. (a) it is not the same as a solution of Eq. (b). With this in mind, the problem with your argument is that in your equation (1) you assume that the Lagrangian is $L_2 = L^2$, and in your equation (2) you then assume that the Lagrangian is $L_1 = L$, which clearly will lead to a problem!






          share|cite|improve this answer












          Let me define these two Lagrangians



          $$
          L_1 = u' tag{a}
          $$



          and



          $$
          L_2 = (u')^2 tag{b}
          $$



          As it turns out $L_1$ has its own equations of motion and they do not need to satisfy equation (b). In other words, a solution of Eq. (a) it is not the same as a solution of Eq. (b). With this in mind, the problem with your argument is that in your equation (1) you assume that the Lagrangian is $L_2 = L^2$, and in your equation (2) you then assume that the Lagrangian is $L_1 = L$, which clearly will lead to a problem!







          share|cite|improve this answer












          share|cite|improve this answer



          share|cite|improve this answer










          answered Nov 24 at 10:03









          caverac

          12.8k21028




          12.8k21028












          • The lagrangian for a geodesic is $sqrt{g_{ij}frac{dx^i}{ds}frac{dx^j}{ds}}$ but they use the square of this quantity as the lagrangian to derive the equations of motion. Shouldn't that be a problem?
            – BinaryBurst
            Nov 24 at 10:09












          • @BinaryBurst Was editing my answer when I saw J.G's
            – caverac
            Nov 24 at 10:20


















          • The lagrangian for a geodesic is $sqrt{g_{ij}frac{dx^i}{ds}frac{dx^j}{ds}}$ but they use the square of this quantity as the lagrangian to derive the equations of motion. Shouldn't that be a problem?
            – BinaryBurst
            Nov 24 at 10:09












          • @BinaryBurst Was editing my answer when I saw J.G's
            – caverac
            Nov 24 at 10:20
















          The lagrangian for a geodesic is $sqrt{g_{ij}frac{dx^i}{ds}frac{dx^j}{ds}}$ but they use the square of this quantity as the lagrangian to derive the equations of motion. Shouldn't that be a problem?
          – BinaryBurst
          Nov 24 at 10:09






          The lagrangian for a geodesic is $sqrt{g_{ij}frac{dx^i}{ds}frac{dx^j}{ds}}$ but they use the square of this quantity as the lagrangian to derive the equations of motion. Shouldn't that be a problem?
          – BinaryBurst
          Nov 24 at 10:09














          @BinaryBurst Was editing my answer when I saw J.G's
          – caverac
          Nov 24 at 10:20




          @BinaryBurst Was editing my answer when I saw J.G's
          – caverac
          Nov 24 at 10:20


















          draft saved

          draft discarded




















































          Thanks for contributing an answer to Mathematics Stack Exchange!


          • Please be sure to answer the question. Provide details and share your research!

          But avoid



          • Asking for help, clarification, or responding to other answers.

          • Making statements based on opinion; back them up with references or personal experience.


          Use MathJax to format equations. MathJax reference.


          To learn more, see our tips on writing great answers.





          Some of your past answers have not been well-received, and you're in danger of being blocked from answering.


          Please pay close attention to the following guidance:


          • Please be sure to answer the question. Provide details and share your research!

          But avoid



          • Asking for help, clarification, or responding to other answers.

          • Making statements based on opinion; back them up with references or personal experience.


          To learn more, see our tips on writing great answers.




          draft saved


          draft discarded














          StackExchange.ready(
          function () {
          StackExchange.openid.initPostLogin('.new-post-login', 'https%3a%2f%2fmath.stackexchange.com%2fquestions%2f3011349%2fcalculus-of-variations-confusion%23new-answer', 'question_page');
          }
          );

          Post as a guest















          Required, but never shown





















































          Required, but never shown














          Required, but never shown












          Required, but never shown







          Required, but never shown

































          Required, but never shown














          Required, but never shown












          Required, but never shown







          Required, but never shown







          Popular posts from this blog

          Le Mesnil-Réaume

          Ida-Boy-Ed-Garten

          web3.py web3.isConnected() returns false always